[FOM] Proof "from the book" of the incompleteness theorem

Martin Davis martin at eipye.com
Tue Aug 24 14:29:54 EDT 2004


How about:

The set of arithmetic theorems of any formal system  is recursively 
enumerable, while the set of arithmetic truths is not. So any sound formal 
system must fail to prove some arithmetic truth.

Martin




More information about the FOM mailing list